LSAT 15 – Section 2 – Question 01

You need a full course to see this video. Enroll now and get started in less than a minute.

Target time: 0:55

This is question data from the 7Sage LSAT Scorer. You can score your LSATs, track your results, and analyze your performance with pretty charts and vital statistics - all with a Free Account ← sign up in less than 10 seconds

Question
QuickView
Type Tags Answer
Choices
Curve Question
Difficulty
Psg/Game/S
Difficulty
Explanation
PT15 S2 Q01
+LR
Sufficient assumption +SA
A
98%
164
B
1%
154
C
1%
147
D
0%
148
E
0%
153
128
135
142
+Easier 144.118 +SubsectionEasier
This page shows a recording of a live class. We're working hard to create our standard, concise explanation videos for the questions in this PrepTest. Thank you for your patience!

This is a sufficient assumption (SA) question, and we know that because of the language in the stem: “Which one of the following principles… would justify…”

Sufficient assumption questions tend to be very formal. We’re looking for a rule that would validate the conclusion, specifically by bridging the premise and conclusion through the rule. Not only are we extrapolating the rule from our argument, but we’re plugging that rule back into the argument to make it “valid.” Our rule/prephrase will look like: if [premise], then [conclusion].

Our first sentence is a prescriptive claim, with a concession: even though smoking is legal, smoking should be banned on flights. The first question we ask is: “Is there a reason this should be the case?” The next sentence attempts to give us an explanation: smoking exposes people to unavoidable harm. Our first sentence is the conclusion, and the second is the explanation/premise.

SA questions often take this form: the conclusion is that something should be done, and our premises attempt to support and validate this prescriptive statement, but it misses the mark. Why? Because the argument makes illicit assumptions.

Back to this stimulus. We could say: “Sure… the harm is unavoidable, but why should we punish the smokers? Why don’t we just prevent non-smokers from flying?”

The good news is that, in trying to make these arguments valid, the solution is simple: bridge your premise and conclusion through a rule: If [premise], then [conclusion]. Our premise here is that “smoking exposes people on planes to unavoidable harm” and our conclusion is that “cigarettes should be banned on all flights.” Our rule will look something like this: “If smoking on planes causes unavoidable harm, then smoking should be banned on all flights.” This is forcing our conclusion to be true by putting the premise and conclusion in a conditional relationship. We’re also trying to make sure that we retain the language/level of certainty in the conclusion when we create our rule. With the rule (or principle, to take the language of the stem) established, the sufficient condition is triggered by the premise and our conclusion is valid. Also a note, the answer choice may not be as straightforward as our prephrase. They could give us the contrapositive of the conditional, or introduce the necessary condition first, so pay attention.

Correct Answer Choice (A) This one perfectly mimics our prephrase. It’s broad, but that doesn’t matter because its scope includes our argument: People should be prohibited from engaging in smoking (or, legal activity) when (or, in those situations in which) that activity will harm people. The order might be confusing to some people, after all, the conclusion is introduced first in the sentence. However, because “when” is a group one indicator, this is correct. If you have trouble understanding this, review conditional logic in the core curriculum.

Answer Choice (B) This is incorrect because of the direction of the conditional statement. The “should be banned” and everything after the group 2 indicator “only if” should be switched. With the language in B, we can’t trigger the conditional and this leaves our argument untouched.

Answer Choice (C) The first half of the answer is pretty good! But “legal activity should be modified?” That’s very ambiguous. Remember, in SA questions, we strive for 100% validity. What would modifying the legal activity of smoking look like? We can’t just assume modifying means banning it.

Answer Choice (D) Read this answer choice very carefully – it’s very wordy, but it’s essentially saying that people should be excluded from situations in which their activity harms others in those situations. With respect to our argument, that would mean smokers should be excluded from flying. This is incorrect because our conclusion claims that smoking should be banned, not smokers.

Answer Choice (E) This is similar to C in that the first half of the conditional is okay. But when we get to the necessary condition, the answer choice pivots. We’re trying to make smoking illegal in airplanes and our answer choice is saying that it should be legal in all situations. This actually weakens our argument.

Take PrepTest

Review Results

Leave a Reply